offan123 är nöjd med hjälpen
offan123 3070
Postad: 11 jan 2023 19:21 Redigerad: 11 jan 2023 19:21

Area

Om det är kring dy, får man då gränserna 0 till 8?

Yngve 37878 – Livehjälpare
Postad: 11 jan 2023 19:40 Redigerad: 11 jan 2023 19:43

Väldigt tydlig och snygg lösning!

Ja, om du ska integrera i y-led så ska gränserna ska vara från y = 0 till y = 8.

Rita gärna "strimlor" som visar hur ett ytelement ser ut. Det hjälper dig att hitta rätt uttryck för integranden.

offan123 3070
Postad: 11 jan 2023 20:15

Förstod inte helt med hur jag skulle göra dessa strimlor. Så dy och dx- integraler ska ju ge samma svar?

Yngve 37878 – Livehjälpare
Postad: 11 jan 2023 20:35 Redigerad: 11 jan 2023 20:38

Dina strimlor ser fina ut.

Två problem:

  • Du skriver att undre gränsen är y = 8 och övre gränsen är y = 0 men det ska vara tvärtom (som du även skrivit i integralen).
  • Du skriver att y=x3y=x^3 innebär att x=3·yx=3\cdot\sqrt{y} Men det stämmer inte. Du bör alltid alltid kontrollera dina omskrivningar innan du fortsätter att räkna. Annars riskerar du att slösa bort värdefull tid.
offan123 3070
Postad: 11 jan 2023 21:01 Redigerad: 11 jan 2023 21:06

Jag löste ut X=tredjerotenur y men skrev det så slarvigt att jag trodde de var 3* rotenur y.

Nu fick jag ut detta fast med minustecken

Yngve 37878 – Livehjälpare
Postad: 11 jan 2023 21:10

Du skriver fortfarande 3y3\sqrt{y} istället för y3\sqrt[3]{y}. Men nu tolkar du det iallafall korrekt som y13y^{\frac{1}{3}}.

Vad tror du själv det beror på att du får ett negativt värde på integralen?

offan123 3070
Postad: 11 jan 2023 21:49

Jag bytte plats på gränserna så det lägsta värdet är där nere och då blir man tvungen att sätta minus framför integralen, men det borde inte bli fel då

Yngve 37878 – Livehjälpare
Postad: 11 jan 2023 21:59 Redigerad: 11 jan 2023 22:01

Du skriver att undre gränsen är 0 och att övre gränsen är 8, vilket är rätt.

Men du sätter upp integralen tvärtom, vilket är fel.

Svara Avbryt
Close